Luke is a locksmith. On weekends he gets paid \$ 64 . 50 per hour, plus a flat rate service fee of \$ 75 for every job. ​ ​On this Saturday he has three separate jobs that will take 1 1/2, 1/4, and 1 1/4 hours to complete respectively

Answers

Answer 1

Answer:

Luke will earn $ 418.50.

Step-by-step explanation:

Given that on weekends Luke gets paid $ 64.50 per hour, plus a flat rate service fee of $ 75 for every job, and on this Saturday he has three separate jobs that will take 1 1/2, 1/4, and 1 1/4 hours to complete respectively, to determine the money you will earn on the day the following calculations must be made:

(75 x 3) + (1.5 x 64.5) + (0.25 x 64.5) + (1.25 x 64.5) = X

225 + 96.75 + 16.125 + 80.625 = X

418.50 = X

So, all Saturday, Luke will earn $ 418.50.


Related Questions

Michael surveyed 960 of the students in his school about their favorite color. 912 students said their favorite color was red. What percentage of the surveyed students said their favorite color was red?

Answers

Answer:

95%

Step-by-step explanation:

912÷960=0.95

Hope this helps.

Find the measure of angle x?

Answers

It’s 43

Since x: 180-47-90

Answer:

x = 43 degrees

Step-by-step explanation:

First, you have to see that there is a right angle of 90 degrees. Since that is there, you know that there is 90 degrees in the bottom and right side as well. So all you do in this question is do 90 - 47 to get your answer of 43 degrees.

Which equation is represented by the graph below?
y=e+5
y=e+4
y = In X+4

Answers

The correct answer is C, y=Inx+4 because the y-intercept is at 4, and the graph depicts the natural log of x

Help!!
what's 7e-9-3e for e=6? ​

Answers

Answer:

15

Step-by-step explanation:

7e (7×6) = 42

42 - 9 = 33

3e (3×6) = 18

33 - 18 = 15

You’re answer is 15. Hope this helps

Correct answer please
I will mark you brainliest for the correct answer
I need this answer asap

Answers

Answer:

Angle 3 and Angle 4

Step-by-step explanation:

Congruent angles are equal to each other, these can't be congruent.

what is the slope of the lines segment with the endpoint (2,-5) and (-8,3)

Answers

Answer: - 0.8
Explanation I subtracted y2 and y1
(3-(-5) = 8
then subtracted x2 and x1
((-8) -2) =-10
Then I divide 8 and -10
And I got -0.8

Find the equation of the curve that passes through the point (x, y) = (0, 0) and has an arc length on the interval x is between 0 and pi over 4 inclusive given by the integral the integral from 0 to pi over 4 of the square root of the quantity 1 plus the cosine squared x, dx . (5 points)

Answers

Answer:

[tex]\displaystyle y=\sin(x),\text{ where } 0\leq x\leq\pi/4[/tex]

Step-by-step explanation:

The curve passes through the point (x, y) = (0, 0) and has an arc length on the interval [0, π/4] given by the integral:

[tex]\displaystyle \int_{0}^{\pi/4}\sqrt{1+\cos^2(x)}\, dx[/tex]

And we want to find the equation of the curve.

Recall that arc length is given by:

[tex]\displaystyle L=\int_a^b\sqrt{1+\Big(\frac{dy}{dx}\Big)^2}\, dx[/tex]

Rewrite our original integral:

[tex]\displaystyle \int_{0}^{\pi/4}\sqrt{1+(\cos(x))^2}\, dx[/tex]

So:

[tex]\displaystyle \frac{dy}{dx}=\cos(x)[/tex]

It follows that:

[tex]\displaystyle y=\sin(x)+C[/tex]

Using the initial condition:

[tex]0=\sin(0)+C\Rightarrow 0=0+C\Rightarrow C=0[/tex]

The equation for our curve is:

[tex]\displaystyle y=\sin(x),\text{ where } 0\leq x\leq\pi/4[/tex]

WILL GIVE BRAINLIEST

Answers

Answer:

it d. the proof demonstrates that a 2 + b 2=c 2

PLEASE HELP due in 8 mins
1/2 (x - 5) = 4/3​

Answers

Answer:   [tex]7\frac{2}{3}[/tex]

Step-by-step explanation:

[tex]\frac{1}{2} (x-5)=\frac{4}{3} \\[/tex]

Multiply both sides by 2

[tex]2(\frac{1}{2} (x-5))= \frac{4}{3} (2)[/tex]

[tex]x-5=\frac{8}{3} \\x-5+5 =\frac{8}{3} +5\\x=7\frac{2}{3}[/tex]

Answer:

1/2(x-5) =4/3

X-5=4/3÷1/2

X-5=8/3

X=8/3+5

X=23/3

Anna is in a cave 50 feet below the cave entrance. She descends 12 feet, then ascends 15 feet. Find her new position relative to the cave entrance. Anna's new position relative to the cave entrance is feet

Answers

Answer:

53

Step-by-step explanation:

50 - 12 = 38

38 + 15 = 53

A Chevy Volt costs $29,600 in the United States. What would it cost in Canada?

Answers

Answer:

I may be wrong, but it should be around 37301.18 Canadian dollars

Step-by-step explanation:

The  Chevy Volt costs  in Canada is 37,888 Canadian Dollar

What is unitary method?

Unitary method is a process by which we find the value of a single unit from the value of multiple units and the value of multiple units from the value of a single unit

What is the current marked value of US dollar to Canadian dollar?

1 United States Dollar = 1.28 Canadian Dollar

According to the question

Chevy Volt costs in the United States  =  $29,600

As, per  current marked value

1 United States Dollar = 1.28 Canadian Dollar

Cost in Canada =  29,600 *  1.28 Canadian Dollar

                         =   37,888 Canadian Dollar

Hence, the  Chevy Volt costs  in Canada is 37,888 Canadian Dollar

To know more about unitary method here:

https://brainly.com/question/19423643

#SPJ2

I really need help with this!!

Answers

Answer:

x = 4

y = 3.5

Step-by-step explanation:

The parallelogram given has opposite sides that are equal in length. Therefore, the diagonals bisect each other into two equal parts respectively.

Thus:

2y = 7

Divide both sides by 2

y = 7/2

y = 3.5

Also,

x + 6 = 10

Subtract 6 from each side

x = 10 - 6

x = 4

Is the relation shown in the table a function? Use the graph and justify your answer. x 2 4 8 10 y 3 2 0 –1 A coordinate grid from 0 to 9 on both axes. A. Yes. The relation is a function. The graph of the function is a straight line and has a constant rate of change. B. Yes. The relation is a function. The graph of the function is a curve. C. No. The relation is not a function. The graph of the relation does not pass through the origin. D. No. The relation is not a function. The graph of the relation is not a straight line.

Answers

Answer:

Yes; for each input there is exactly one output. 2. ANSWER: No; the domain value 6 is paired with both 9 and 10. 3. {(2, 2), (−1, 5), (5, ... Draw a graph showing the relationship between

Step-by-step explanation:

i think that is the answer

Answer: Yes, the relatoin is a function. the graph of the function is a straight line.

Step-by-step explanation: If it is a straight line it is a function.

There are 20 boys in Mrs. Shaw’s 3rd-period class. 11 of the boys are from Benignus. What percent of the boys came from Benignus?

please hurry
6th grade math

Answers

11/20 times 100% = 55%

HELP ILL GIVE BRAINIEST THIS MY LAST QUESTION!

Jim and Regina can complete one job in 4 hours when working together. When working alone, Jim works twice as quickly as Regina.

• How long will it take each of them to do the job when working alone? Show all work supporting your answer.

• If Regina works as quickly as Jim, how long will it take both of them to do the job working together? Show all work supporting your answer.

Answers

Answer:

Jim = 6 hoursRegina = 12 hoursTogether at same rate = 3 hours

Step-by-step explanation:

Let Jim's speed  is J and Regina's speed is R and the work is W

We have:

(J + R)*4 = WJ = 2R

Substitute J with 2R:

(2R + R)*4 = W12R = W

Regina needs 12 hours to complete work

and

Substitute R with J/2:

(J + J/2)*4 = W 6J = W

Jim needs 6 hours to complete same work

If Regina works as quickly as Jim:

J + R = 2J

Then

Jim can complete 1/6 of work in one hour, they both can complete twice work:

1/6*2 = 1/3

Time to complete work:

1: 1/3 = 3 hours

Answer:

Honestly, Mark above person answer as brainliest..okay

What is the value of n in the equation (2n + 4) + 6 = -9 + 4(2n

+

1)?

(Edulastic)

Answers

Answer:

5/2

Step-by-step explanation:

(2n+4)+6=-9+4(2n+1)

2n+4+6=-9+8n+4

2n+10=-9+8n+4

2n-8n=-9+4-10

-6n=-5-10

-6n=-15

6n=15

n=15/6

simplify

n=5/2

winston works 40 hours a week as a pharmacist. if he made 75,400 last year, how much was he paid per hour?

Answers

Answer:

If I'm calculating correctly, it might be $36.15. Or, if you're rounding, $36.2.

Step-by-step explanation:

There are about 52.14 weeks in a year. If you multiply 40 by 52.14 you get 2085.6. Then you divide 75,400 by 2085.6 and that's how I got my answer. If I'm wrong, someone feel free to correct me.

Answer:

$36.25

Step-by-step explanation:

Ape.x

|x-(-12)| if x>-12


ill give brainlyest if corect

Answers

Answer:

x+12

Step-by-step explanation:

:)

what does x= in each of the questions

Answers

Answer:

x+35/5=2

x+35=10

x=-25

Answer:

[tex]\frac{1}{5} x[/tex]+7=2, x=-25

[tex]\frac{m}{7} -7=3[/tex], m= 105

[tex]\frac{3}{5} x+7=-20, x= -45[/tex]

[tex]-\frac{2}{7} t+5=17, t=-42[/tex]

4.9x+4.2=9.1, x=1

Explanation for last two:

[tex]-\frac{2}{7}t+5=17[/tex]  

Step 1: Subtract 5 from both sides.

[tex]\frac{-2}{7} t+5-5=17-5[/tex]

[tex]\frac{-2}{7} t=12[/tex]

Step 2: Multiply both sides by 7/(-2).

[tex](\frac{7}{-2} )*( \frac{-2}{7}t )= (\frac{7}{-2} )*(12)[/tex]

t=−42

4.9x+4.2=9.1

Step 1: Subtract 4.2 from both sides.

4.9x+4.2−4.2=9.1−4.2

4.9x=4.9

Step 2: Divide both sides by 4.9.

[tex]\frac{4.9x}{4.9} \frac{4.9}{4.9}[/tex]

x=1

Use 3.14 for pi and round to the nearest tenth.

Answers

Answer:

3.1

Step-by-step explanation:

if the last number is less thank five round down

How many cups are in 1 quart?

correct=brainliest

Answers

Answer: 1 quart = 4 cups or 2 pints

Evan is going to invest in an account paying an interest rate of 6.1% compounded continuously. How much would Evan need to invest, to the nearest ten dollars, for the value of the account to reach $67,000 in 7 years?

Answers

Answer:

43720

Step-by-step explanation:

It was right on delta math

please help!! I'm really confused

Answers

Answer:

a. 7/16

b. 27.5%

Step-by-step explanation:

add all parts together to get a total of 80 students. then divide the number of the correct area by 80. part a would be 43.75% and part c would be 27.50%

pls help i need to pass

Answers

Answer:

The answer would be A

Step-by-step explanation:

the answer is A because it shows the correct set of data

The area of a
circular fountain is
225πft2. What is the
diameter of the
fountain?

Answers

Using the formula for area of a circle: area = pi x r^2

You now have:

225pi ft^2 = pi x r^2

Divide both sides by pi:

225 = r^2

Take the square root of both sides:

r = 15

The radius is 15 feet.

The diameter is the radius x 2:

Diameter = 15 x 2 = 30 feet.

Answer: 30 feet.

Mr. Dietrch's class is 17/25 boys. What percentage of the class is boys? 

Answers

Answer:

I believe it's 44% could be wrong tho sorry-

What is the first step Anika made from the original equation.​

Answers

Answer:

She factored out a 4 from the equation.

Step-by-step explanation:

So our original equation is 12x + 4 = 20x - 12

Anika's first steep of her equation is 3x + 1 = 5x - 3

We know that the terms were not moved around, so the first step was likely multiplication or division.

We also see that a 4 has been factored out of the entire equation, like this:

12x + 4 = 20x - 12

4(3x + 4) = 4(5x - 3)

And these 4's cancel each other out.

So, I conclude that Anika's first step was dividing the entire equation by 4 or factoring out a 4 from the equation.

The skateboard ramp has a slope of 2/5 if the height is 6 what is the width?

Answers

Answer:

its  5y−30

         2

Step-by-step explanation:

Because lets put this problem in slope form. The form is Y=mx+b

ok now lets put it with the number Y=2/5+6


What is the area of this figure?
Enter your answer in the box

Answers

area of square is a^2
5^2 =25
area of triangle is base x height divide by 2
5x4=20
20 divide by 2 is 10
add them together
25+10=35

ABCD is a parallelogram. If mZCDA = 81, then
mZDAB = ? _. The diagram is not to scale.

Answers

Answer:

∠DAB = 99°

Step-by-step explanation:

Given:

ABCD is  PARALLELOGRAM

∠CDA = 81°

Find:

∠DAB

Computation:

ABCD is  PARALLELOGRAM

So,

∠CDA + ∠DAB = 180°

81° + ∠DAB = 180°

∠DAB = 99°

Other Questions
If you want 2 cookies and you only have.50 cents and the cookies cost 10 dollars how much money will you have to get if you only have.50 cents AOT FANS WHO READ ON AO3 so like i am here reading a levi ff and i have music playing it Slumber party starts playing, i lost it, perfect song and i laughed because i saw another ff of bottom levi so i couldn't see it the same. i needed to share that. Which of the following statements is true about cells and their organelles?A. Cells are living and the organelles that make them up are also living.B. Cells are nonliving and the organelles that make them up are living.C. Cells are living and the organelles that make them up are nonliving.D. Cells are nonliving and the organelles that make them up are also nonliving. PLZ HELP I DIDNT MEAN TO CLICK THAT ANSWR I NEEDD HELP did americans have the right to manifest destiny?fake answers will be reported. is anyone like good at math that can help me real quick? Suddenly she felt the scallop contract in her hand. Its shell clamped shut, and watersquirted out of it. Right into her face.What is the main function of this text in the narrative? (5 points)1) Climax2) Exposition3) Falling actionO4) Resolution 6. Which of the following is most likely to be a role in a retail career?O A. MechanicO B. BabysitterO C. BuyerO D. Therapist How do I find the solution to 1 = r /12 ? The sum of the angles triangle are 180 degrees Use that to find the m What does the mode represent in a set of numbers? how did switzerland become a french speaking country? What does the transition word ESPECIALLY do in writing? ver resposta das fraes 1/53/7 7 inches, 8 inches, 9 inches will these lengths form a triangle write 24 minutes to 3 hours as a ratio in its simplest form Write a complete sentence using the following information and the verb combination tener que. El muchacho/beber agua. 17. Please type the correct conjugation for each verb. Make sure your answers are in lower caseincluding the first letter. The verb is escribirlos estudiantes(1 Point)Enter your answer I will give you brainilest.You might think that the angle between the hour and minute hand at 2:20 is 60 degrees, but it is not! The hour hand has moved beyond the 2. Calculate the angle between the clock hands at 2:20. I need help pls its on edg and I have no clue Nnnxndndndnxndndndnndndnd